Help on LR question--I thank you ahead of time 4 your input Forum

Prepare for the LSAT or discuss it with others in this forum.
Post Reply
User avatar
LSAT_Padawan

New
Posts: 45
Joined: Thu Feb 18, 2010 10:17 pm

Help on LR question--I thank you ahead of time 4 your input

Post by LSAT_Padawan » Mon Feb 22, 2010 12:40 am

Why is "A" the correct answer choice to the following question stated below?

Proponents of the electric car maintain that when the technical problems associated with its battery design are solved, such cars will be widely used and, because they are emission-free, will result in an abatement of the environmental degradation caused by auto emissions. But unless we dam more rivers, the electricity to charge these batteries will come from nuclear or coal-fired power plants. Each of these three power sources produces considerable environmental damage. Thus, the electric car____.
Which one of the following most logically completes the argument?

(A) will have worse environmental consequences than its proponents may believe
(B) will probably remain less popular than other types of cars
(C) requires that purely technical problems be solved before it can succeed
(D) will increase the total level of emissions rather than reduce it
(E) will not produce a net reduction in environmental degradation

I picked (E) because I incorrectly surmised that the stimulus conclusion states "but unless we dam more rivers, the electricity to charge these batteries will come from nuclear or coal-fired power plants. Each of these three power sources produces considerable environmental damage. Thus, the electric car_____" will not produce a net reduction in environmental degradation. I guess that is NOT 100% true because the they can figure out a clean way (damming rivers) than the electricity will not come from dirty power sources that cause enviro damage. The correct answer is (A) will have worse environmental consequences than its proponents may believe. But I can argue that the premise states that if they figure out a way to dam the rivers, it will produce clean electricity so it will NOT "have worse environmental consequences."


User avatar
bees

Bronze
Posts: 310
Joined: Tue Oct 20, 2009 11:50 pm

Re: Help on LR question--I thank you ahead of time 4 your input

Post by bees » Mon Feb 22, 2010 1:06 am

Aren't you having to add a premise in order to make your argument against (A)? You're not allowed to do that. As far as the stimulus is concerned, more dams aren't being built, so the electricity is coming from other sources, resulting in (A) being the correct answer.

Also, what the above poster said.

spearnreel

New
Posts: 99
Joined: Wed Oct 07, 2009 9:30 pm

Re: Help on LR question--I thank you ahead of time 4 your input

Post by spearnreel » Mon Feb 22, 2010 1:10 am

Damming rivers also produces considerable environmental damage.

WellNow

New
Posts: 18
Joined: Fri May 01, 2009 7:22 pm

Re: Help on LR question--I thank you ahead of time 4 your input

Post by WellNow » Mon Feb 22, 2010 1:16 am

All three sources produce considerable environmental damage. Also, E is not correct because the prompt does not tell you that an increase in the use of dirty power sources will entirely offset the environmental benefits of reduced emissions. Just because all 4 produce negative environmental consequences does not mean that they do so in equal measures.

Want to continue reading?

Register now to search topics and post comments!

Absolutely FREE!


User avatar
haole_20

New
Posts: 35
Joined: Wed Jan 06, 2010 3:09 pm

Re: Help on LR question--I thank you ahead of time 4 your input

Post by haole_20 » Mon Feb 22, 2010 1:25 am

The choices clearly come down to "A" and "E". I would choose "A" primarily because it is a weaker statement. Stronger stances are tougher to adequately support. I'm sure there is a better reason for why "A" is correct but this is a method I often used on these types of questions (especially when narrowed down to 2 choices) and it served me well.

EDIT: and by weaker I mean it uses the word "may"

Shrimps

Bronze
Posts: 269
Joined: Sun Feb 14, 2010 10:04 pm

Re: Help on LR question--I thank you ahead of time 4 your input

Post by Shrimps » Mon Feb 22, 2010 2:36 am

First of all, what does this question have to do with LSAT? "Which one of the following most logically completes the argument?" This is a GMAT-type question, isn't it?

"(E) will not produce a net reduction in environmental degradation" is plainly wrong because it makes a claim that can only be proven with statistics, which the text does not offer.

"(A) will have worse environmental consequences than its proponents may believe" can be correct in that it attacks the beliefs of the proponents, not actual facts. Problem is, the author of the text never attempted to prove that the proponents have unrealistic expectations of the environmental consequences of switching to electricity-powered vehicles. This text can be examined as an example of an informal fallacy, a straw-man argument, a type of an ad hominem or something along those lines. Ask people better trained in informal logic than me.

Anyway, LSAT_Padawan: there are close to 60 real LSAT tests available for you, plus some excellent prep materials using those, like the two Powerscore bibles. Why are you doing these sorts of questions?

skip james

Bronze
Posts: 262
Joined: Sat Sep 19, 2009 2:53 am

Re: Help on LR question--I thank you ahead of time 4 your input

Post by skip james » Mon Feb 22, 2010 10:41 am

Shrimps wrote:First of all, what does this question have to do with LSAT? "Which one of the following most logically completes the argument?" This is a GMAT-type question, isn't it?

"(E) will not produce a net reduction in environmental degradation" is plainly wrong because it makes a claim that can only be proven with statistics, which the text does not offer.

"(A) will have worse environmental consequences than its proponents may believe" can be correct in that it attacks the beliefs of the proponents, not actual facts. Problem is, the author of the text never attempted to prove that the proponents have unrealistic expectations of the environmental consequences of switching to electricity-powered vehicles. This text can be examined as an example of an informal fallacy, a straw-man argument, a type of an ad hominem or something along those lines. Ask people better trained in informal logic than me.

Anyway, LSAT_Padawan: there are close to 60 real LSAT tests available for you, plus some excellent prep materials using those, like the two Powerscore bibles. Why are you doing these sorts of questions?
this is a real question. it's from one of the rare tests (1-6,8,17), if i remember correctly.

skip james

Bronze
Posts: 262
Joined: Sat Sep 19, 2009 2:53 am

Re: Help on LR question--I thank you ahead of time 4 your input

Post by skip james » Mon Feb 22, 2010 10:43 am

Shrimps wrote:First of all, what does this question have to do with LSAT? "Which one of the following most logically completes the argument?" This is a GMAT-type question, isn't it?
oh and this question type is essentially a 'most strongly supported' question, just phrased a bit differently.

Want to continue reading?

Register for access!

Did I mention it was FREE ?


User avatar
blhblahblah

Bronze
Posts: 132
Joined: Mon Aug 27, 2007 10:54 pm

Re: Help on LR question--I thank you ahead of time 4 your input

Post by blhblahblah » Mon Feb 22, 2010 11:45 am

Not a rare test, but between PT 9-12, IIRC.

Shrimps

Bronze
Posts: 269
Joined: Sun Feb 14, 2010 10:04 pm

Re: Help on LR question--I thank you ahead of time 4 your input

Post by Shrimps » Mon Feb 22, 2010 12:01 pm

The egg on my face is just breakfast leftovers, nothing else. LSAT did evolve, huh.

skip james

Bronze
Posts: 262
Joined: Sat Sep 19, 2009 2:53 am

Re: Help on LR question--I thank you ahead of time 4 your input

Post by skip james » Mon Feb 22, 2010 1:36 pm

Shrimps wrote:The egg on my face is just breakfast leftovers, nothing else. LSAT did evolve, huh.
no they still do this shit. you know those annoying fill-in-the-blank questions?

like: blah blah blah, thus __________________.

Which of the following logically completes the argument?

same shit.

Shrimps

Bronze
Posts: 269
Joined: Sun Feb 14, 2010 10:04 pm

Re: Help on LR question--I thank you ahead of time 4 your input

Post by Shrimps » Mon Feb 22, 2010 11:18 pm

So, how frequent are these questions on the latest tests (49-59)?

Register now!

Resources to assist law school applicants, students & graduates.

It's still FREE!


Post Reply

Return to “LSAT Prep and Discussion Forum”